LSAT and Law School Admissions Forum

Get expert LSAT preparation and law school admissions advice from PowerScore Test Preparation.

User avatar
 Dave Killoran
PowerScore Staff
  • PowerScore Staff
  • Posts: 5853
  • Joined: Mar 25, 2011
|
#26372
Complete Question Explanation
(The complete setup for this game can be found here: lsat/viewtopic.php?t=6413)

The correct answer choice is (E)


If Q is given in the morning, the only morning that can occur is Wednesday morning. Additionally, from the first rule J must be given with K. From the second rule, R must be given with N or S, and that block must be placed on Thursday or Friday. This presents a problem for Wednesday afternoon: only Q, R, or S can be given on Wednesday afternoon, but Q is given Wednesday morning, and R is given Thursday or Friday afternoon. Thus, only S can be given on Wednesday afternoon, leaving N to be given on the same day as R. This information results in the following setup:
powerscore_M12_T3_J2010_LG_explanations_game_1_#5_diagram_1.png
Accordingly, S cannot be given on Thursday, and answer choice (E) cannot be true and is correct.
You do not have the required permissions to view the files attached to this post.
 deck1134
  • Posts: 160
  • Joined: Jun 11, 2018
|
#49295
Why is it not possible to have N/S as a block instead of RN, and then get a completely different answer? That is what happened to me on this question.
 deck1134
  • Posts: 160
  • Joined: Jun 11, 2018
|
#49296
Nevermind! I forgot the restriction on R by rule #2.

Get the most out of your LSAT Prep Plus subscription.

Analyze and track your performance with our Testing and Analytics Package.